Answer these for points

Answer These For Points
Answer These For Points

Answers

Answer 1

6. f(x)g(x)+h(x) = 20x3-4x. This answer is arrived at by using the rules of algebra and the values of the given functions.

7. f(x)g(x) = -15x2 - 18x - 15

What are polynomials?

Polynomials are algebraic expressions consisting of variables and coefficients that are combined through addition, subtraction, multiplication and division operations. For example, the polynomial 4x² + 3x - 5 can be written as the sum of 4x², 3x and -5.

6. In order to calculate f(x) g(x)+h(x), it is necessary to first multiply f(x) and g(x). Since f(x)=4x and g(x) = 5x2-4, it follows that f(x)g(x)=20x3-4x. Next, it is necessary to add h(x) to this product. Since h(x) = 9x, it follows that f(x)g(x)+h(x) = 20x3-4x+9x = 20x3-4x. Since each coefficient is correctly represented in the answer, it follows that the correct answer is 20x3-4x.

Multiplying f(x) and g(x) gives the product of 20x3-4x, and adding h(x) to this product gives the sum of 20x3-4x. As each coefficient is correctly represented in the answer, the correct answer is 20x3-4x.

7. The product of two polynomials is found by multiplying each term of one polynomial by each term of the other polynomial. This is known as the FOIL method (First, Outer, Inner, Last).

The first term is the product of the first terms of each polynomial, which is 3x x 4x = 12x2.

The second term is the product of the outer terms, which is 3x x -5x = -15x2.

The third term is the product of the inner terms, which is 5 x -5x = -25x.

Finally, the fourth term is the product of the last terms of each polynomial, which is 5 x -3 = -15.

We can now combine the terms to get the product of the two polynomials, which is:

f(x)g(x) = -15x2 - 18x - 15.

Therefore, the correct coefficient for each term is -15x2 for the first term, -18x for the second term, and -15 for the third term.

For more questions related to algebra

https://brainly.com/question/28036476

#SPJ1


Related Questions

An isosceles right triangle is removed from
each corner of a square piece of paper, as
shown, to create a rectangle. If AB = 12 units,
what is the combined area of the four removed
triangles, in square units?

Answers

The combined area of the four removed triangles is 48 sq.units. Answer: 48

We need to find out the combined area of the four removed triangles, in square units. Given: AB = 12 units.

Let's consider the given square, and let's draw an altitude BD and also draw perpendiculars to BD from the three vertices A, C and D.

Let AB = x cm. Area of square = x² sq.cm.

Now, we are cutting a triangle with base x and height x, which is a right-angled triangle. Hence, area of each removed triangle = (1/2) * x * x = (x²/2) sq.cm.

Now, BD = x/√2. Area of rectangle = AB * BD = 12 * 12/√2 = 72√2 sq.cm.

Now, area of 4 triangles = (x²/2) + (x²/2) + (x²/2) + (x²/2) = 2x² sq.cm.

We know that, Area of rectangle = Area of 4 triangles + Area of square => 72√2 = 2x² + x² => 72√2 = 3x² => x² = 24√2 cm² => x = √(24 * 2) cm = √(48) cm = 4√3 * √2 cm.

Area of 4 triangles = 2x² sq.cm = 2 * 24 cm² = 48 sq.cm.

Hence, the combined area of the four removed triangles is 48 sq.units. Answer: 48.

To learn more about area, refer below:

https://brainly.com/question/27683633

#SPJ11

Suppose the central perk coffee shop sells a cup of espresso for two dollars in a cup of cappuccino for $2. 50 on Friday. Destiny sold 30 more cups of cappuccino then espresso for a total of $178. 50 worth of espresso and cappuccino how many cups of each month old

Answers

Destiny sold 23 cups of espresso and 53 cups of cappuccino on Friday, for a total sales amount of $178.50.

Let's start by defining our variables. Let x be the number of cups of espresso sold and y be the number of cups of cappuccino sold. We know that the price of espresso is $2 per cup and the price of cappuccino is $2.50 per cup. We also know that Destiny sold 30 more cups of cappuccino than espresso and the total sales amount was $178.50.

Using this information, we can create a system of equations to solve for x and y. The first equation comes from the fact that Destiny sold 30 more cups of cappuccino than espresso:

y = x + 30

The second equation comes from the total sales amount:

2x + 2.5y = 178.5

Now we can substitute the first equation into the second equation and solve for x:

2x + 2.5(x + 30) = 178.5

2x + 2.5x + 75 = 178.5

4.5x = 103.5

x = 23        

So Destiny sold 23 cups of espresso. Using the first equation, we can find that she sold 53 cups of cappuccino:

y = x + 30

y = 23 + 30    

y = 53

Therefore, Destiny sold 23 cups of espresso and 53 cups of cappuccino on Friday, for a total sales amount of $178.50.

To know more about total sales amount click here:

brainly.com/question/13571917

#SPJ4

I need help with dis math

Answers

answer:
3s + 7.99 = 71.83

the answer is that because u have three identical so 3s as s is shirts and u add the 7.99 as a fee and it all equals to 71.83

What is the simplest form of 8(5k+7)−10(6k−7)

Answers

The simplest form of the given expression is -20k + 126.

To find the simplest form of the expression 8(5k+7)−10(6k−7), follow these steps:
1. Distribute the numbers outside the parentheses to the terms inside the parentheses:
  8 × 5k + 8 × 7 - 10 × 6k + 10 × 7
2. Perform the multiplication:
  40k + 56 - 60k + 70
3. Combine like terms (terms with the same variable and exponent):
  (40k - 60k) + (56 + 70)
4. Simplify the expression by performing the subtraction and addition:
  -20k + 126
The simplest form of the given expression is -20k + 126.

for such more question on simplest form

https://brainly.com/question/344788

#SPJ11

at 3:25 p.m., two trains left kalamazoo, michigan. one train traveled westward at a constant rate of

Answers

When they are 111 miles apart, the current time is 4:10 p.m.

If the time taken for two trains to be apart by 111 miles by "t" time.

Then at that time, t, the train which is at 82 mph, should have traveled a distance of 82T miles.

d1 = 82t

At the same time, t, the train which is at 66 mph, should have traveled a distance of 66T miles.

d2= 66t

The total distance traveled by both trains is d1 + d2 = D

D = 82t + 66t

D = 148t

From the given data, D = 111

Hence, 148t = 111

Solving the equation, the time t

t = 111÷148

t = 0.75

Converting it into minutes,

t = 0.75*60

t = 45 mins.

The current time ( T ) = the time at which both the trains left ( t1 ) + 45 mins.

T = t1+t

T = 3:25 + 45 min.

T =4:10 pm

Therefore, the current time is 4:10 p.m

To know about Time traveled problems:

https://brainly.com/question/2854969

The complete question is-

At 3:25 p.m., two trains left Kalamazoo, Michigan. one train traveled westward at a constant rate of 82 miles per hour, while the other traveled eastward at a constant rate of 66 miles per hour. if they are now 111 miles apart, what time is it now? show your work on how you solved this situation.

When the temperature drops below 15°C in a building, the furnace turns on.
At what temperatures will the heater turn on? Write an inequality to represent
this situation, and graph the solution on a number line.

Answers

The inequality to represent this situation is T < 15°C, where T is the temperature.

What is inequality?

Inequality is a statement that two values, expressions, or quantities are not equal. Inequality is usually represented by the symbols ">", "<", "≥", or "≤".

This inequality can be graphed on the number line by representing 15°C as a point on the number line. Any values to the left of 15°C, such as 14°C, 13°C, and so on, would be represented as points to the left of 15°C on the number line.

Less than inequality is used to compare two values ​​to see if one is less than the other. In this case, the inequality T < 15°C states that the temperature T must be less than 15°C in order for the furnace to turn on.

Graphically, the solution to this inequality is represented by a number line with a point at 15°C and all points to the left of 15°C represented in the solution set.

For more questions related to number line

https://brainly.com/question/25230781

#SPJ1

PACKAGING A video game system is packaged in a box that is in the shape of a cube. The length of the packaging box is 4x^2 y^5 . What is the volume of the packaging box in terms of x and y?

Answers

Answer: 64x^6y^15

Step-by-step explanation:

If the packaging box is in the shape of a cube, then all its sides are equal in length. Let's call the length of each side "s".

We know that the length of the packaging box is 4x^2 y^5, so:

s = 4x^2 y^5

To find the volume of the packaging box, we need to calculate s^3 (since the box is a cube).

s^3 = (4x^2 y^5)^3

s^3 = 4^3 (x^2)^3 (y^5)^3

s^3 = 64x^6 y^15

Therefore, the volume of the packaging box in terms of x and y is 64x^6 y^15.

26. In the given figure, OP || RS. ZPQR = 60° and QRS = 130°. Then what is the measure of ZOPQ? S P 60% R 130⁰​

Answers

Answer: The answer is 60.

Step-by-step explanation:

Using the fact that OP || RS, we know that∠RWV = 180° − 130° 1.  ∠RWV = 50° We know that,∠PWQ = ∠RWV = 50° (Since, opposite angles of intersecting lines are equal) Also, for line OP∠OQP + θ = 180° θ = 180° − ∠OPQ = 180° − 110° 2.  θ = 70° 

Answer:

The measure of ∠OPQ is 110°.

Step-by-step explanation:

Draw a line parallel to OP from point Q. Label a point on the line T. (See attached diagram).

Angles SRQ and TQR are alternate interior angles, and so according to the Alternate Interior Angles Theorem, they are congruent.

⇒ m∠TQR = m∠SRQ = 130°

Given m∠PQR = 60° and m∠TQR = 130° then:

⇒ m∠TQP + m∠PQR = m∠TQR

⇒ m∠TQP + 60° = 130°

⇒ m∠TQP = 70°

Angles OPQ and TQP are same-side interior angles, and so according to the Same-side Interior Angles Theorem, they are supplementary (sum to 180°).

⇒ m∠OPQ + m∠TQP = 180°

⇒ m∠OPQ + 70° = 180°

⇒ m∠OPQ = 110°

Therefore, the measure of ∠OPQ is 110°.

the average of 8 numbers is 14. the average of 6 of those numbers is 12. what is the average of the other two numbers?

Answers

The average of the other two numbers is 16.

This is because the average of 8 numbers is 14, and the average of 6 of those numbers is 12. Therefore, to find the average of the remaining two numbers, you need to find the difference between the average of 8 and the average of 6.

The difference between 14 and 12 is 2. Therefore, if the average of 6 is 12, the average of the remaining two numbers must be 12 + 2 = 16.

To solve this question, you need to use the fact that the average of 8 numbers is 14, and the average of 6 of those numbers is 12. The average of 8 numbers is the sum of all 8 numbers divided by 8. Therefore, if the average of 8 numbers is 14, the sum of all 8 numbers must be 14x8 = 112.

Similarly, the average of 6 numbers is 12, so the sum of those 6 numbers must be 12x6 = 72. The difference between these two sums is 112-72 = 40. This is the sum of the remaining two numbers.

Therefore, if the sum of the remaining two numbers is 40, then the average of the remaining two numbers must be 40/2 = 20.

To know more about average click on below link:

https://brainly.com/question/24057012#

#SPJ11

Factorise p²q-pr²-pq+r²

Answers

p²q - pr² - pq + r²

p²q - pq - pr² + r²


pq(p - 1) - r²(p - 1) (

(pq - r²)(p - 1)


The factorized form of the expression is:
(pq - r²)(p - 1).

if you're good at quadratics...

Answers

Therefore, the correct answer is:   [tex](x+10)^2=82[/tex]. ( right hand side of the equation).

What is equation?

An equation is a mathematical statement that indicates that two expressions are equal. It typically contains variables, which are represented by letters, and may also include constants and operators. The general format of an equation is:

expression = expression

For example, the equation x + 2 = 6 means that the expression x + 2 is equal to the expression 6. The goal of solving an equation is to find the value(s) of the variable(s) that make the equation true.

To solve the quadratic equation by completing the square, Jamie needs to follow the steps:

Move the constant term to the right-hand side of the equation:

[tex]x^2 + 20x = -18[/tex]

Add and subtract the square of half the coefficient of x to the left-hand side of the equation:

[tex]x^2 + 20x + (20/2)^2 - (20/2)^2 = -18[/tex]

[tex](x+10)^2 - 100 = -18[/tex]

Simplify the right-hand side of the equation:

[tex](x+10)^2 = 82[/tex]

To know more about coefficient, visit

https://brainly.com/question/30066987

#SPJ1

Prove that the following statement is false. There exists an integer n such that 6n2 + 27 is prime. To prove the statement is false, prove the negation is true. Write the negation of the statement. For every integer n, 6n² + 27 is prime. For every integer n, 6n2 + 27 is not prime. There exists an integer n, such that 6n2 + 27 is not prime. There exists a composite number q = 6n2 + 27, such that n is an integer. There exists an integer n, such that 6n2 + 27 is prime. Now prove the negation. Suppose n is any integer. Express 6n2 + 27 as the following product: 6n2 + 2 Now is an integer because sums and products of integers are integers. Thus, 6n2 + 27 is not prime because it is a

Answers

The negation of the statement "There exists an integer n such that 6n2 + 27 is prime" is "For every integer n, 6n2 + 27 is not prime."

To prove the negation, we can use algebraic manipulation to show that 6n2 + 27 is always composite.

Suppose n is any integer. We can factor out 3 from 6n2 + 27 to get 3(2n2 + 9). Since 2n2 + 9 is always odd (2 times any integer is even, and adding 9 makes it odd), we can further factor it as (2n2 + 9) = (2n2 + 6n + 9 - 6n) = [(2n+3)(n+3)] - 6n.

Substituting this expression back into 3(2n2 + 9), we get 3[(2n+3)(n+3) - 6n]. Since (2n+3)(n+3) - 6n is an integer, 3[(2n+3)(n+3) - 6n] is composite for every integer n. Therefore, 6n2 + 27 is not prime for any integer n.

For more questions like Integer click the link below:

https://brainly.com/question/490943

#SPJ11

A triangle with side lengths 7, 6, 4 is

Acute

Right

Obtuse

Answers

Right

so 7 is the hypotenuse because it is the biggest. so you have to use 6 and 4 in the formula to see if they equal 7.

(a)²+(b)²=c²

(6)²+(4)²=c²

36+16=c²

(square root) 52=c²

the square root of 52 is 7

so therefore it is a right triangle.

what is the future value of 6000 earning 18% interest, compounded monthly for 8 years

Answers

Answer:

To calculate the future value of an investment earning compound interest, we can use the formula:

FV = P(1 + r/n)^(nt)

where:

FV is the future value

P is the principal (starting amount)

r is the annual interest rate (as a decimal)

n is the number of times the interest is compounded per year

t is the number of years

In this case, we have:

P = 6000

r = 0.18 (18% annual interest rate)

n = 12 (compounded monthly)

t = 8

Substituting these values into the formula, we get:

FV = 6000(1 + 0.18/12)^(12*8)

FV = 6000(1.015)^96

FV = 6000(3.045)

FV = 18270

Therefore, the future value of $6000 earning 18% interest, compounded monthly for 8 years, is $18,270.

what is the value of t?

Answers

Answer:

t=36°

Step-by-step explanation:

90-54=36

opposite angles are equal so t=36°

Which of the following is a point of tangency on the circle below?
A. Point Y
B. Point Z
C. Point W
D. Point X

Answers

Based on the diagram, the point of tangency is Point Z. Therefore, the answer is B.

What is point of tangency?

A point of tangency is a point at which a straight line, called a tangent line, touches a curve or a circle at only one point.

At the point of tangency, the tangent line is perpendicular to the radius of the circle that intersects the point of tangency.

The point of tangency is the point where a tangent line to the circle touches the circle at only one point.

In the given diagram, the line segment XZ appears to be a tangent to the circle.

Therefore, the point of tangency is the point where the line segment XZ touches the circle.

To know more about tangent line visit:

https://brainly.com/question/30162653

#SPJ9

given the following frequency table of values, is the mean, median, or mode likely to be the best measure of the center for the data set? valuefrequency 351 364 376 386 395 631

Answers

For the given following frequency table of values 351, 362, 373, 381, 391, The mode is likely to be the best measure of the center for the data set.

The given frequency table is as follows:

        Value frequency 351, 362, 373, 381, 391.

        To find the most appropriate measure of central tendency for a dataset, we need to analyze the spread of data.

The mean, median, and mode are measures of central tendency in statistics.

We can find the following measures from the given data set:

       Mean: It is calculated by summing up all the values and then dividing the result by the total number of values. This measure of central tendency is appropriate when the data are symmetrical.

      Median: It is the middle value of the data set when arranged in order. It is suitable for skewed data.

      Mode: It is the most common value in the data set. It is appropriate when data is discrete. The data in the frequency table appear to be discrete.

      Because the data are discrete, the most appropriate measure of central tendency is the mode. So, the mode is likely to be the best measure of the center for the given value frequency data set.

Learn more about the mean, median, and mode at: https://brainly.com/question/14532771

#SPJ11

consider the differential equation given by[math equation]the goal of this problem is to solve this differential equation numerically, analytically and compare the solutions. find the exact solution (i.e. the analytical solution) use euler's method to solve the differential equation with a step size h=0,001; (this is the numerical solution)

Answers

The number of iterations increases. If there is a significant difference between the two solutions, we may need to investigate the numerical method used or check for errors in our analytical solution.

Step-by-step explanation:

The differential equation is missing in your question. However, I will give a general overview of how to solve a differential equation numerically using Euler's method and how to find an analytical solution.

Numerical Solution using Euler's Method:

Suppose we have a first-order differential equation of the form y' = f(x, y), where y' represents the derivative of y with respect to x. To solve this numerically using Euler's method, we need to start with an initial condition y(x0) = y0, and we want to find the value of y at some other point x1 = x0 + h.

The Euler's method involves approximating the derivative y' by the difference quotient (y1 - y0) / h, where y1 is the value of y at x1. Rearranging this equation, we get:

y1 = y0 + h * f(x0, y0)

Using this equation, we can iteratively compute the value of y at different points by using the previous value of y. For example, to find y2, we can use the equation:

y2 = y1 + h * f(x1, y1)

We continue this process until we reach the desired endpoint.

Analytical Solution:

An analytical solution to a differential equation is an explicit expression for y(x) that satisfies the differential equation for all values of x. To find an analytical solution, we may use techniques such as separation of variables, integrating factors, or other methods specific to the type of differential equation.

For example, if we have a differential equation of the form y' = k * y, where k is a constant, we can use separation of variables to obtain:

dy / y = k * dx

Integrating both sides, we get:

ln|y| = k * x + C

where C is an arbitrary constant of integration. Solving for y, we get:

y = Ce^(kx)

where C = ±e^C is a constant determined by the initial condition.

Comparison of Solutions:

Once we have the numerical and analytical solutions, we can compare them by plotting the graphs of y(x) for each method. If the numerical solution was computed with a small enough step size, it should converge to the analytical solution as the number of iterations increases. If there is a significant difference between the two solutions, we may need to investigate the numerical method used or check for errors in our analytical solution.

To learn more about approximating visit:

https://brainly.com/question/30945002

#SPJ11

x^2-3x-40=0 solve for x

Answers

Answer:

Step-by-step explanation:

x^2-3x-40=0

x^2-3x=40

2x-6x=40

-4x=40

-4x/4 = 40/-4

x= -10

Answer:

x=8 or x=-5

Step-by-step explanation:

x²-3x-40=0

x²-8x+5x-40=0

x(x-8)+5(x-8)=0

(x-8)(x+5)=0

⇒x=8 or x=-5

QUESTION 1. Assume we are testing a function with 3 variables:
Variable A: has values 0 and 1
Variable B: has values 0 and 1
Variable C: has values 0 and 1
What is the total 2-way variable value configuration coverage achieved by the following tests:
A=0; B=0; C=1
A=0; B=1; C=1
A=1, B=0, C=0

Answers

The total 2-way variable achieved by the given tests is 6.

How to find 2-way variable?

There are three pairs of variables, and each pair can have two possible values, resulting in 2-way variable value configurations. Therefore, the total 2-way variable value configuration coverage achieved by the given tests is 6, as follows:

A=0, B=0

A=0, C=1

B=0, C=1

A=0, B=1

A=1, B=0

A=1, C=0

Learn more about 2-way variable

brainly.com/question/29435977

#SPJ11

the volume of a cube decreases at a rate of 6 m 3 / s . find the rate at which the side of the cube changes when the side of the cube is 4 m . answer exactly or round to 2 decimal places.

Answers

The rate at which the side of the cube changes when the side of the cube is 4 m is -1/8 m/s (or approximately -0.125 m/s).

Let's use the formula for the volume of a cube:

V = s³

where V is the volume and s is the length of one side of the cube. To find the rate of change of the side length, we need to differentiate this formula with respect to time t:

dV/dt = d/dt (s³) = 3s² ds/dt

We know that dV/dt = -6 m³/s (the negative sign indicates that the volume is decreasing), and when s = 4 m, we have:

-6 = 3(4²) ds/dt

Simplifying this equation gives us:

ds/dt = -6 / (3*4²) = -1/8 m/s

Learn more about cube here: brainly.com/question/30962206
#SPJ4

if you have $11 and save $5 each week how much money you will have after 6 weeks

Answers

Answer: 41$

Step-by-step explanation:

This is because 5x6=30 (To find how much money is made)

then 11+30=41 (add both amounts)

Pls just say a b c or d

Answers

I believe it will be the answer B because there are the three possibilities

if one flag pole is y feet tall and casts a shadow x feet long, then how tall is another nearby flag pole that casts a shadow p feet long at the same time of day?

Answers

If one flag pole is y feet tall and casts a shadow x feet long, and another nearby flag pole casts a shadow p feet long at the same time of day, we can use similar triangles to determine the shadow of the second flag pole.

In this scenario, the two flag poles and the ground form two similar right triangles. The height of the first flag pole (y) corresponds to one leg of the first triangle, and the length of its shadow (x) corresponds to the other leg.

Similarly, the height of the second flag pole (h) corresponds to one leg of the second triangle, and the length of its shadow (p) corresponds to the other leg.

Therefore, the height of the second flag pole is equal to the product of the height of the first flag pole and the length of the shadow of the second flag pole, divided by the length of the shadow of the first flag pole.

For more details about shadow click here:

https://brainly.com/question/31162142#

#SPJ11

a high school baseball player has a 0.253 batting average. in one game, he gets 8 at bats. what is the probability he will get at least 6 hits in the game?

Answers

The probability of a high school baseball player getting at least 6 hits in one game, given a 0.253 batting average, when he gets 8 at-bats, is 0.0197 or approximately 2%.


Given, the high school baseball player's batting average is 0.253, which means in 100 times he hits the ball, he will make 25.3 hits on average. We need to find the probability of getting at least 6 hits in a game when he gets 8 at-bats.

We will calculate the probability using the Binomial Probability formula. Here, the number of trials is 8, and the probability of success is 0.253. We need to find the probability of getting at least 6 hits.

P(X≥6) = 1 - P(X<6)

P(X<6) = ∑P(X=i), i=0 to 5

We can use the Binomial Probability Table to find these probabilities or use the Binomial Probability formula.

P(X<6) = P(X=0) + P(X=1) + P(X=2) + P(X=3) + P(X=4) + P(X=5)

= C(8,0) (0.253)^0 (1 - 0.253)^8 + C(8,1) (0.253)^1 (1 - 0.253)^7 + C(8,2) (0.253)^2 (1 - 0.253)^6 + C(8,3) (0.253)^3 (1 - 0.253)^5 + C(8,4) (0.253)^4 (1 - 0.253)^4 + C(8,5) (0.253)^5 (1 - 0.253)^3

≈ 0.9799

Therefore, P(X≥6) = 1 - 0.9799

= 0.0201 or approximately 2%.

Hence, approximately 0.0197 or 1.97% is the probability of a high school baseball player, who has a batting average of 0.253, obtaining at least 6 hits when given 8 at-bats during a single game.

To know more about probability refer here:

https://brainly.com/question/1596693#

#SPJ11

Given f(x)=5x+7 and g(x)=2x+2, find g(g(1-3w))

Enter as the final value or expression without parentheses

Answers

As a result, the final number or expression is g(g(1-3w)) ≈ -12w + 10 (without parenthesis).

Which of these are they known as?

When adding extraneous information or perhaps an afterthought to a sentence, parentheses, a pair or punctuation marks, are most frequently utilized. Two curving vertical lines can be seen in parentheses: ( ).

We must first evaluate g(1-3w) and then re-insert that result into g(x) in order to determine g(g(1-3w)).

We must first determine g(1-3w):

Substitute x with 1-3w to get g(x) ≈ 2x + 2 and g(1-3w) ≈ 2(1-3w) + 2.

g(1-3w) ≈ 2 - 6w + 2  (distribute the 2)

g(1-3w) ≈ -6w + 4 (combine similar terms) (combine like terms)

We can again again enter the result of g(1-3w) into g(x):

If you substitute g(1-3w) for x, then g(x) ≈ 2x + 2 g(g(1-3w)) ≈ 2(-6w Plus 4) + 2

g(g(1-3w)) ≈ -12w + 8 + 2 (allocate the 2) (distribute the 2)

g(g(1-3w)) ≈ -12w + 10 (combine comparable terms) (combine like terms)

To know more about parentheses visit:

https://brainly.com/question/28146414

#SPJ1

a sample of test scores is normally distributed with a mean of 120 and a standard deviation of 10. what score is located 2 standard deviations below the mean? g

Answers

The score located 2 standard deviations below the mean is 100. This score can be found by subtracting 2 standard deviations (20) from the mean (120).

The normal distribution is a bell-shaped curve that is symmetrical around the mean. This means that if you calculate the number of standard deviations away from the mean, you can use the same number to calculate how many standard deviations away from the mean the score is.

For example, in this question, the mean is 120 and the standard deviation is 10. To find the score located 2 standard deviations below the mean, subtract 2 standard deviations from the mean. This means the score is 120 - 20 = 100.

In general, the formula for calculating the score located x standard deviations away from the mean is:

Score = Mean + (x * Standard Deviation)

For example, to find the score located 4 standard deviations away from the mean, the formula is:

Score = Mean + (4 * Standard Deviation)

In this example, the score is 120 + (4 * 10) = 160.

In summary, to find the score located x standard deviations away from the mean, use the formula:
Score = Mean + (x * Standard Deviation)

See more about normal distribution at: https://brainly.com/question/14243195

#SPJ11

Translate the figure 5 units left and 5 units up. -10-9 Plot all of the points of the translated figure. You may click a plotted point to delete it.​

Answers

I hoped this helped!  

: )

Step-by-step explanation:

originally - (6,-1)     After translation - (1,4)

originally - (8,-1)     After translation - (3,4)

originally - (4,-7)     After translation - (-1,-2)

originally - (7,-9)     After translation - (3,-4)

originally - (9,-9)     After translation - (4,-4)

What is the endpoint of a line segment with these points? Endpoint: Z(–21, 15) Midpoint: M(–13, 29) (–5, 43) (–17, 22) (–27, 21) (–29, 1)

Answers

To find the endpoint of the line segment with endpoint Z(–21, 15) and midpoint M(–13, 29), we need to use the midpoint formula, which states that the midpoint of a line segment is the average of its endpoints:

Midpoint formula: (x1 + x2)/2, (y1 + y2)/2 = (x, y)

We can plug in the values we know:

(x1 + x2)/2 = -13
(y1 + y2)/2 = 29
x1 = -21
y1 = 15

Solving for x2 and y2:

x2 = -2x1 - 26 = -2(-21) - 26 = 16
y2 = 2y1 - 18 = 2(15) - 18 = 12

Therefore, the endpoint of the line segment with endpoint Z(–21, 15) and midpoint M(–13, 29) is E(16, 12).

Answer: A - (-5, 43)

Step-by-step explanation:

A business owner applies for a credit card to cover $14,000 in emergency expenses. The credit card charges 16.99% annual interest compounded continuously. If no payments are made for 2 years, what will the balance on the card be, rounded to the nearest penny?

Answers

Credit card charges $19665.33 will the balance on the card be, rounded to the nearest penny.

What is interest in simple words?

When you borrow money, you must pay interest, and when you lend money, you must charge interest. The most common way to represent interest is as a percentage of a loan's total amount per year. The interest rate for the loan is denoted by this proportion.

                                  Interest is the cost of borrowing money and is typically stated as a percentage, such an annual percentage rate (APR). Lenders may charge interest to borrowers for the use of their funds, or borrowers may charge interest to lenders for the use of their funds.

amount applied for = $14,000

interest rate = 16.99%

the balance after 2 years

             P₀ = $1400

             r = 16.99% = 0.1699

             t = 2

        [tex]P_{0} = P_{0}e^{rt}[/tex]

   [tex]P_{2} = 1400e^{0.1699 * 2}[/tex]

         ≈ $19665.33

Learn more about Interest

brainly.com/question/30393144

#SPJ1

             

Other Questions
Which industry would most likely not benefit from the applications of biotechnology? Compare5. Review lines 135140 in Scene 4. What does Macbeth realize, and in what way does this speech reveal a change in Macbeths attitude from how he has felt in the past about his deeds? Select ALL the set of numbers that are possible values for x in the inequality, x who is credited with the idea of stacking increasingly smaller mastabas to create a monumental symbol of the king's everlasting power? group of answer choices khufu djoser khafre imhotep a restaurant is open 24 days each month. its monthly fixed payroll cost is $31,800 and its daily variable payroll cost is $2,100. what is the restaurant's total daily payroll cost? $2,000 $3,800 $3,425 $4,200 a 50-year-old woman has experienced devastating consequences in her family and work life as a result of her longstanding alcohol addiction. her levels and pathways of which neurotransmitter are likely to differ from an individual's without addiction? help pls its too much who believes that gentle birth decends from ancient wealth? a the wife of bath b the franklin c the knight d the crone what does it mean to have a conflict style? how does culture influence our conflict style? provide at least three examples. on january 1, nees manufacturing lends $10,000 to roberson supply using a 9% note due in eight months. calculate the amount of interest revenue nees will record on september 1, the date that the note is due. you are going to receive $5,000 at the end of each year for four years. if the annual discount rate is 7%, what is the present value of this income stream? an object falls freely from rest on a planet where the acceleration due to gravity is 20 m/s2. after 5 seconds, the object will have a speed of use all of the given elements to write a complete sentence indicating where people are going. use the appropriate present-tense form of the verb ir and make any necessary changes? please help immediately please go to my profile and answer the other I need them asap. can you identify the minerals that comprise igneous rocks? drag the appropriate labels to their respective targets. the specific colors of the referenced mineral crystals within the rocks have been supplied to assist you. labels will only be used once, and not all labels will be used.\ in the absence of chromosomal rearrangements, what are the most likely karyotypes of a newborn baby with 47 chromosomes? with 45 chromosomes? When calculating a feeder or service load for existing installations, if the maximum demand data for a 1-year period is not available, the calculated load shall be permitted to be based on the maximum demand (the _____ kilowatts reached and maintained for a 15-minute interval) continuously recorded over a minimum 30-day period using a recording ammeter or power meter connected to the highest loaded phase of the feeder or service, based on the initial loading at the start of the recording.highest average220.87(1) Ex study the photograph (source 4). who is depicted, and how were they posed? what does this suggest about the victory garden program as well as war efforts on the homefront more broadly? simone can effectively identify the best possible answers to multiple-choice test questions. this best what did one dog say to the other dog